Question Summary
0 of 66 questions completed
Questions:
- 1
- 2
- 3
- 4
- 5
- 6
- 7
- 8
- 9
- 10
- 11
- 12
- 13
- 14
- 15
- 16
- 17
- 18
- 19
- 20
- 21
- 22
- 23
- 24
- 25
- 26
- 27
- 28
- 29
- 30
- 31
- 32
- 33
- 34
- 35
- 36
- 37
- 38
- 39
- 40
- 41
- 42
- 43
- 44
- 45
- 46
- 47
- 48
- 49
- 50
- 51
- 52
- 53
- 54
- 55
- 56
- 57
- 58
- 59
- 60
- 61
- 62
- 63
- 64
- 65
- 66
Information
Start Exam
You have already completed the Exam before. Hence you can not start it again.
Exam is loading...
You must sign in or sign up to start the Exam.
You have to finish following Exam, to start this Exam:
Results
0 of 66 questions answered correctly
Your time:
Time has elapsed
You have reached 0 of 0 points, (0)
Categories
- Not categorized 0%
- 1
- 2
- 3
- 4
- 5
- 6
- 7
- 8
- 9
- 10
- 11
- 12
- 13
- 14
- 15
- 16
- 17
- 18
- 19
- 20
- 21
- 22
- 23
- 24
- 25
- 26
- 27
- 28
- 29
- 30
- 31
- 32
- 33
- 34
- 35
- 36
- 37
- 38
- 39
- 40
- 41
- 42
- 43
- 44
- 45
- 46
- 47
- 48
- 49
- 50
- 51
- 52
- 53
- 54
- 55
- 56
- 57
- 58
- 59
- 60
- 61
- 62
- 63
- 64
- 65
- 66
- Unanswered
- Answered
- Review
-
Question 1 of 66
1. Question
A 47 year old female presents with a history of a direct blow over the shaft of her left humerus. Her x-ray shows a spiral fracture of the distal third of the humerus. On examination of her hand, sensation is intact on the palmar side but absent on the dorsum of her hand. Which one of the following nerves is most likely injured?
Correct
Radial nerve
Incorrect
Explanation:
The radial nerve provides cutaneous innervation to the dorsum of the hand. As such, an injury to that nerve can cause deficient sensation on the dorsum (back of) the hand. Because she has intact sensation on the palmar surface of her hand, her medial and ulnar nerves are not likely injured.
-
Question 2 of 66
2. Question
Prior to performing a peripheral nerve block of the median, nerve at the level of the wrist, you determine that the anesthetic effect should last for at least 2 hours. The best agent for this, purpose is
Correct
Bupivacaine (Marcaine), 0.25% plain
Incorrect
Explanation:
Bupivacaine has an expected duration of action of 2-4 hours. Procaine, lidocaine, and chloroprocaine have durations of action ranging from 15 minutes to 1 hour.
-
Question 3 of 66
3. Question
A patient following a car accident presents with bruising of the eyelid and clear fluid is observed leaking from his nose. Which one of the following you suspect as a cause?
Correct
Anterior cranial fossa fracture
Incorrect
Explanation:
Anterior cranial fossa fractures are associated with CSF rhinorrhea and bruising around the eyes, ie, “raccoon eyes.”
Defects in the floor of the anterior cranial fossa can lead to the leakage of CSF into the nasal cavity. Because of the microbiologic nature of the nasal cavity and its continuity with the rest of the upper aerodigestive tract, communication with the intracranial contents can lead to meningitis and other intracranial complications, including serious and sometimes lethal medical problems.
With increased performance of endoscopic procedures of the anterior skull base and given the nature of trauma to this area, the number of patients with CSF rhinorrhea has increased. -
Question 4 of 66
4. Question
An 18 year old woman suffered a motor vehicle accident and is brought to emergency department. On examination, she has a Glascow coma scale (GCS) of 10 and swelling over the occipital protuberance. The appropriate imaging study in this patient would be?
Correct
Head CT without contrast
Incorrect
Explanation:
In suspected intracranial bleeds it is important to do a CT of the head without contrast. The reason is because both contrast and blood look white on a CT image. Therefore contrast is not indicated in the imaging T of the head, during an episode of likely intracranial bleeding or hemorrhage.
CT of the head with contrast is done when a tumor or intracranial mass is suspected, for example in a patient that presents with headache, vomiting and papilledema. -
Question 5 of 66
5. Question
A 29 year old man has been in rehabilitation for C6 complete quadriplegia. He had a dicing accident 2 months ago that caused paralysis. His recovery has been steady. His vital signs have been normal and stable.
The nurse calls and tells you that for the past hour the patient has experienced sweating, rhinorrhea, and a pounding headache. His heart rate is 55/min and his blood pressure is 220/115 mm Hg. His temperature and respirations are reported as normal. No nausea or vomiting is present and his nervous system examination is unchanged. The most likely diagnosis isCorrect
Autonomic hyperreflexia
Incorrect
Explanation:
Autonomic hyperreflexia is characterized by the sudden onset of headache and hypertension in a patient with a lesion above the T6 level. There may be associated bradycardia, sweating, dilated pupils, blurred vision, nasal stuffiness, flushing, or piloerection. It usually occurs several months after the injury and has an incidence as high as 85% in quadriplegic patients. Frequently, it subsides within 3 years of injury, but it can recur at any time. Bowel and bladder distension are common causes. Hypertension is the major concern because of associated seizures and cerebral hemorrhage.
Cluster headaches have a constant unilateral orbital localization. The pain is steady (nor-throbbing) and lacrimation and rhinorrhea may be part of the syndrome. Sepsis is usually manifested by chills, fever, nausea, and vomiting. Common signs include tachycardia and hypotension rather than bradycardia and hypertension. Signs and symptoms of intracranial hemorrhage vary depending upon the site of the hemorrhage, but the unchanged neurologic status and the lack of a history of hypertension decrease the likelihood of this diagnosis. There are no neurologic findings or history which suggests progression of the patient´s lesion at C6. -
Question 6 of 66
6. Question
A 70 year old male has known coronary artery disease, presents with complains of pain in his back and legs. The pain aggravated by standing and walking and relieved by sitting. On examination, deep tendon reflexes in his legs are 0 to 1+ bilaterally. He has mild muscle weakness of his quadriceps. He is taking ibuprofen, 800 mg three times a day, and using a back brace without much relief. His symptoms would be relieved by which one of the following?
Correct
Posterior spinal decompression surgery
Incorrect
Explanation:
This patient´s symptoms are typical of spinal stenosis, as they are present when he is standing and relieved by sitting. He has already failed a trial of h3AIDs and bracing. Spinal decompression surgery is now indicated. Epidural corticosteroid injection might be helpful, but not trigger-point injections. Oral corticosteroids would be helpful if he had a herniated disc. Sympathectomy and bypass surgery are treatments for vascular occlusion.
-
Question 7 of 66
7. Question
A woman aged 56 is unable to dorsiflex her foot five weeks after the removal of a cast. Which one of the following nerve is most likely injured?
Correct
Common peroneal
Incorrect
Explanation:
The peroneal nerve is a branching of the sciatic nerve, which supplies movement and sensation to the lower leg, foot and toes. Trauma to the nerve can result in a condition called foot drop, where dorsiflexion of the foot is compromised and the foot drags during walking and sensory loss to the dorsal surface of the foot and portions of the anterior, lower-lateral leg.
-
Question 8 of 66
8. Question
Pallidotomy is a surgical therapy for
Correct
Parkinson´s disease
Incorrect
Explanation:
Thalamotomy and pallidotomy, contralateral to the side of the body that is most affected, are most effective for the treatment of disabling unilateral tremor and dyskinesia from Parkinson´s disease.
-
Question 9 of 66
9. Question
Thalamotomy and Pallidotomy are surgical procedures for which of the following?
Correct
Parkinson´s disease
Incorrect
Explanation:
Thalamotomy and pallidotomy, contralateral to the side of the body that is most affected, are most effective for the treatment of disabling unilateral tremor and dyskinesia from Parkinson´s disease.
-
Question 10 of 66
10. Question
A young man, during a fight, sustains a severe blow to the head. X-ray of the skull reveals air in the cranial vault. Which one of the following is represented by this finding?
Correct
Skull fracture
Incorrect
Explanation:
Pneumocephalus is the presence of air or gas within the cranial cavity. Pneumocephalus (ie, subdural air causing mass effect on the brain) is caused by conditions that lead to increased air pressure within the subdural space. Causes of pneumocephalus include skull base fracture, transsphenoidal surgery, tumors involving the skull base and congenital skull base defects.
-
Question 11 of 66
11. Question
A patient has a subarachnoid hemorrhage caused by a right anterior communicating artery aneurysm. He undergoes successful surgery 2 days after the hemorrhage. At third post-operative day right arm weakness develops. The most likely diagnosis is
Correct
Vasospasm
Incorrect
Explanation:
Ischemic symptoms from vasospasm occur in about one-third of all patients with subarachnoid hemorrhage. The risk of developing vasospasm can often be predicted by the amount of blood present on a CT scan obtained one day after hemorrhage. Some doctors will give nimodipine to prevent vasospasm in these patients.
-
Question 12 of 66
12. Question
A 22-year-old had an accident when he was not wearing his seat belt. On physical exam, he had ecchymosis of the left external ear, blue-gray discoloration behind the mastoid process, and bilateral peri-orbital ecchymosis (raccoon eyes). He also had unilateral hearing loss and anosmia. What type of injury did he suffer from?
Correct
Basilar skull fracture
Incorrect
Explanation:
Basilar skull fractures are part of temporal bone fractures and can be detected by drain-age of cerebrospinal fluid coming out of the nares. “Raccoon” eyes and “Battles” sign (blue-gray discoloration behind the mastoid process) are also present. Frontal sinus fractures usually result from a dashboard injury from a motor vehicle accident. It is associated with severe frontal headaches and epistaxis. A CT scan is diagnostic. Nasal fractures result in epistaxis, tenderness, and crepitations of nasal bones. A septal hematoma may be found. Orbital floor “blow-out” fracture are very serious. They are secondary to blunt anteroposterior force directed against the eyeball. Diplopia and restricted upward gaze from ocular muscle entrapment occur. Transverse fractures are more serious injury to the temporal bone, resulting in complete sensorineural hearing loss and facial nerve injury.
Basilar skull fracture involves the temporal bone, occipital bone, sphenoid bone, and/or ethmoid bone. It is not equivalent to middle cranial fossa fracture. Racoon eyes may be bilateral or unilateral. If bilateral, it is highly suggestive of basilar skull fracture, with a positive predictive value of 85%. Most often associated with fractures of the anterior cranial fossa, it occurs as blood from skull fracture seeps into the soft tissue around the eyes.
-
Question 13 of 66
13. Question
All of the following are contraindications for the execution of a lumbar puncture EXCEPT
Correct
Infection of either or both kidneys.
Incorrect
Explanation:
The lumbar puncture is used to obtain cerebrospinal fluid for the diagnosis of a number of disease states including meningitis, encephalitis, multiple sclerosis, Guillain-Barre syndrome, and others. The contraindications to performing a lumbar puncture include mass lesion of the CNS, suspected occlusion of the venous sinus, increased intracranial pressure, and hemorrhagic diathesis. Infection of the site of the lumbar puncture may be a contraindication to performing it; however, infection at other sites does not contraindicate the lumbar puncture.
-
Question 14 of 66
14. Question
Which of the following cardiac lesions does not produce ischemic stroke?
Correct
Coronary atherosclerosis
Incorrect
Explanation:
Approximately two-thirds of ischemic strokes are accounted for by atherosclerosis of the extracranial and intracranial arteries. Cerebral emboli from cardiac sources account for about one-third of ischemic strokes. These include embolization of mural thrombi in cardiomyopathies. Mechanical heart valves carry a higher risk of embolization than prosthetic valves. Embolization from cardiac myxomas occurs from either overlying thrombus or from the tumor itself. Both acute and subacute endocarditis produces cardiac vegetations which can embolize. Although coronary atherosclerosis is associated with disease of other vessels (myocardial infarction and atrial fibrillation any of which may produce stroke), it does not by itself produce ischemic stroke.
-
Question 15 of 66
15. Question
A 17 year old boy presents with headache and alterations in mental status. Imaging confirms pontine mass. Surgery is initiated and a frozen section sent for examination describes the lesion as “a CNS neoplasm containing bizarre pleomorphic tumor cells accompanied by necrosis and hemorrhage.” The most likely diagnosis is
Correct
Glioblastoma multiforme
Incorrect
Explanation:
Glioblastoma multiforme is the most aggressive tumor in the astrocytic line. Histologically, the tumor tends to show necrosis and hemorrhage on biopsy or cut section. The tumor cells are obviously pleomorphic, and frequently are bizarre in appearance with markedly enlarged, abnormal nuclei. Glioblastoma multiforme tends to be a tumor of the cerebral cortex in adults and of the brain stem in children. Ependymomas in this age group, commonly arise from the fourth ventricle. These present as papillary masses, characteristically with perivascular rosettes. Fibrillary astrocytoma represents a well differentiated variant of astrocytic tumors, whereas glioblastoma multiforme is a poorly differentiated one.
Oligodendrogliomas and pilocytic astrocytomas do not usually show pleomorphism, necrosis, or hemorrhage. -
Question 16 of 66
16. Question
A 24 year old man undergoes magnetic resonance angiography (MRA) of the brain because of symptoms suggestive of a berry aneurysm. Which vessel in the circle of Willis that is the most frequent site of berry aneurysm?
Correct
Anterior cerebral artery
Incorrect
Explanation:
Berry aneurysm is a variant of aneurysm that occurs in the intracranial cavity. It develops as a saccular expansion of the arterial wall and most commonly affects the anterior part of the circle of Willis, including the anterior cerebral artery (ACA) close to where the anterior communicating artery branches off. The two ACAs proceed parallel to each other in a posterior anterior direction, connected by the short anterior communicating artery. The ACA then courses around the genu (the “knee”) of the corpus callosum and distributes tributaries to the medial surface of the cerebral hemisphere the basilar artery can be seen opposite to the ACA (down in the picture) as it bifurcates to give rise to the paired posterior cerebral arteries (PCA). The basilar artery results from fusion of the two vertebral arteries at the level of the caudal pons. Berry aneurysms arising from the posterior part of the circle of Willis are less common than the ones in the anterior part. The most common site of posterior berry aneurysms is near the origin of the posterior inferior cerebellar artery (PICA). The middle cerebral artery (MCA) is seen as a thick vessel that runs laterally on both sides toward the temporal lobe. It represents the main source of blood for the lateral cerebral hemisphere. The ophthalmic artery is a branch of the internal carotid artery. It runs anteriorly and provides blood supply to the orbital content. This vessel cannot be appreciated here.
-
Question 17 of 66
17. Question
Presence of a pituitary tumor is most likely indicated by which of the following?
Correct
Enlarged sella turcica associated with bony erosion
Incorrect
Explanation:
Of all of these possibilities, only the combination of an enlarged sella turcica and bony erosions would be highly suggestive of a pituitary tumor. Enlargement of the sella turcica on radiograph by itself can be seen without pituitary tumors. However, bony erosions increase the probability of a pituitary tumor. The other answers suggest abnormal activity in the thyroid, adrenal cortex, blood vessels or musculature, or the presence of one of several conditions that can increase the melanin content of the skin (e.g. Addison´s disease).
-
Question 18 of 66
18. Question
A 42 year old man suffered cervical hyperextension injury or whiplash during a road traffic accident. A neurological exam and cervical X rays ruled out any significant neurological injury. Which of the following is NOT commonly prescribed treatment for whiplash?
Correct
Use of traction.
Incorrect
Explanation:
No studies to date have demonstrated the benefit of traction compared to placebo, and it is recommended that physical therapy, such as increased range of motion exercises, begin as soon as the patient can tolerate them. Warm compress and hot showers are beneficial in decreasing stiffness. A soft cervical neck collar may help alleviate muscle spasm.
-
Question 19 of 66
19. Question
An individual underwent a surgery to remove areas of the brain in an effort to control his epilepsy. Following operation the patient exhibited highly amnestic responses for the remainder of his life. Part of the brain was most likely removed is
Correct
Temporal lobe
Incorrect
Explanation:
Removal of the temporal lobe was formerly performed to control epilepsy. This often caused amnestic responses so severe that a patient may recognize a face on Tuesday, then on Wednesday deny that he has ever seen the face. Surgical removal of none of the other areas of the brain listed will result in amnestic responses.
-
Question 20 of 66
20. Question
A 30-year-old man is stabbed in the neck He has left-sided hemiparesis with complete loss of discriminatory joint position and vibration sensation below C8 on the left side. On the right side, there is loss of pain and temperature sensation below C8. What findings can be expected from an MRI?
Correct
A hemisection of the left side of the cord
Incorrect
Explanation:
A hemisection of the cord produces Brown-Sequard syndrome. Below the level of the lesion there is ipsilateral hemiparesis, ipsilateral, loss of discriminatory touch sensation, and contralateral loss of pain and temperature sensation. A hemisection of the left half of the spinal cord will transect the fibers of the corticospinal tract on the left side, resulting in left-sided motor paralysis. (These motor fibers originate in the contralateral cortex, decussate in the medulla, and descend in the lateral funiculus of the spinal cord.) The hemisection will also transect the ascending fibers of the dorsal column pathway that carry sensory information concerning discriminatory touch, vibration, and joint position from the same side. (This tract will subsequently cross in the medulla and end in the contralateral cerebral cortex.) In contrast, the fibers carrying information about pain and temperature decussate segmentally in the spinal cord and form the spinothalamic tract, which ascends in the lateral funiculus. Damaging the spinothalamic tract on the left side impairs sensation arising from the right side of the body.
A complete transection of the spinal cord will cause loss of sensation and motor function bilaterally below the transection. Transection of the dorsal columns on the left side will interfere only with fine touch, vibration, and joint-position sense. A lesion of the lateral funiculus will produce deficits in motor function and pain and temperature sensation but not in discriminatory touch. Lesions of the cervical dorsal roots on the left side will produce anesthesia to discriminatory touch, as well as pain and temperature on the left side. Only the dermatomal areas supplied by those roots will be affected. Such lesions will not produce hemiparesis because motor fibers exit the spinal cord in the ventral roots. -
Question 21 of 66
21. Question
A newly born baby is found to have a prominent defect at the back of his spine through which his meninges and spinal cord protrude. A failure of which of the following processes is the most common cause of this type of defect?
Correct
Fusion of the vertebral arches
Incorrect
Explanation:
The condition described is spina bifida with myelomeningocele. A failure of the posteriorly located vertebral arches to fuse posteriorly causes spina bifida, which can vary in severity from a completely asymptomatic minor abnormality to protrusion of the spinal cord and roots through an open skin defect, with a very real risk of minor trauma or infection causing paralysis.
The bodies of the vertebrae are the stacking ovoid structures on the anterior aspect of the spinal canal. The pedicles attach the bony ring that surrounds the spinal canal to the body of the vertebrae. Failure of development of one of the paired primary ossification centers of the body can produce very severe scoliosis. The superior articular process articulates with the inferior articular facet of the vertebra above it. -
Question 22 of 66
22. Question
A patient presents with a complete paralysis following an injury to the spinal cord four hours ago. Which of the following medications should be started?
Correct
Dexamethasone
Incorrect
Explanation:
Steroids have been shown to be of benefit in reducing the level of injury if started within eight hours of injury to the spinal cord. Indomethacin and aspirin both have anti inflammatory effects, but neither has been shown, to be of any benefit in spinal cord injuries. Angiotensin exerts its primary effects on the renal and vascular system. It plays no role in the treatment of spinal cord injury. Altered seratonin levels have been implicated as the cause for a variety of psychiatric and mood related disorders, but it has not been shown to be of any benefit in the treatment of spinal cord injuries.
-
Question 23 of 66
23. Question
A 46 year old man develops a large meningioma that compresses the brain. CT scan of head shows a subfalcine herniation. Which of the following structures would most likely be damaged by herniation?
Correct
Cingulate gyrus
Incorrect
Explanation:
Brain herniation can occur either with generalized increased intracranial pressure or as a consequence of a mass lesion of the cranium. Subfalcine herniation occurs when part of one cerebral hemisphere herniates under the falx membrane, which separates the two cerebral hemispheres along the midline of the skull. The gyrus most affected is the cingulate gyrus, which runs along the medial aspect of the cerebral hemisphere, just above the corpus callosum. The cerebellar tonsils can herniate into the foramen magnum, in a tonsillar herniation. The medulla can be compressed by the herniating cerebellar tonsils, producing damage to this structure. The medial temporal lobe can be damaged in uncal (transtentorial) herniation. The midbrain can be damaged in the course of a uncal (transtentorial) herniation.
-
Question 24 of 66
24. Question
A 52 year old female underwent neurosurgery for resection of an intracranial neoplasm attached to the dura. The specimen is shown in the below. The tumor compressed the underlying brain parenchyma without infiltration. The most likely diagnosis is
Correct
Meningioma
Incorrect
Explanation:
The gross features of this dural based tumor are consistent with meningioma the most frequent benign intracranial neoplasm. It arises from meningothelial cells and histologically consists of whorls of elongated cells with scattered psammoma bodies. The benign behavior of this tumor is apparent from its “pushing” pattern of growth. The tumor tends to expand downward, compressing the brain, but without: invading it. For this season, this tumor can be easily removed at surge. An arteriovenous malformation is a vascular aggregate of arteries, veins, and vessels with intermediate characteristics. These lesions are usually intracerebral and manifest with hemorrhage or seizures. Glioblastoma multiforme is the most frequent malignant intracerebral tumor. It arises from neoplastic transformation of astrocytes within the white matter. Grossly, it is characterized by a variegated appearance with areas of solid tumor alternating with necrosis and hemorrhage.
Medulloblastoma develops from the cerebellar vermis, usually in children. It is composed of small, primitive appearing neoplastic cells. Oligodendroglioma is a neoplasm of oligodendroglial cells. It develops within the white matter, often in close proximity to the neocortex, and frequently manifests with seizures. Neoplastic cells closely resemble normal oligodendrocytes. -
Question 25 of 66
25. Question
An 8-year-old boy lost consciousness after falling off his skateboard. At the scene, he had stable pulse and blood pressure with 95% saturation and fingerstick glucose of 101 mg/dL. He is not arousble to any stimuli. Pupils are reactive. Rest examination is normal. He has several abrasions on his forearms and legs, and there is soft-tissue swelling at his occiput. No significant lab findings exist. CT shows a hypodense lesion within the pons, with a hyperdense focus within the basilar artery consistent with thrombus. What is the next step in the management of this patient?
Correct
Angiography of the neck vessels
Incorrect
Explanation:
This patient has a basilar thrombus resulting in acute brain-stem infarction. In an 8-year-old patient who has just sustained trauma, the leading diagnosis is dissection of one or both of his vertebral arteries with distal embolization. Angiography of the neck can confirm this diagnosis.
Hypercoagulable workup can be reserved for circumstances where the angiogram does not show a dissection.
Lumbar puncture will likely be unhelpful because the patient r does not have an acute inflammatory process as the etiology of his loss of consciousness.
Transthoracic and transesophageal echocardiogram while providing data about a cardiogenic embolic source, are unlikely to reveal the diagnosis, inasmuch as the acute thrombus is following a known trauma. -
Question 26 of 66
26. Question
A 38-year-old woman had a head-on collision with a tractor-trailer. On arrival, she is seatbelt but unconscious. She is intubated and intravenous Ringers lactate commenced. Pupils are fixed and dilated bilaterally. There is no response to painful stimuli. The only other injuries are a right-sided pneumothorax and a separated right shoulder, which are already appropriately managed. There are no obvious injuries to the face or scalp. A CT scan of the head shows diffuse blurring of the grey-white matter interface and multiple small, punctuate hemorrhages. What is the next most appropriate step in the management of the patient´s neurologic state?
Correct
Attempt to lower intracranial pressure without surgery
Incorrect
Explanation:
This patient´s physical and radiographic findings are consistent with diffuse axonal injury. This occurs in severe trauma and CT scan findings include diffuse blurring of the gray-white matter interlace with multiple small punctate hemorrhages. Without hematoma, there is no role for surgery. Therapy is directed at keeping the intracranial pressure (ICP) low to prevent further damage. This includes head elevation, hyperventilation, and avoidance of fluid overload. An ICP monitor may also be used (this can be placed at the bedside). Mannitol, furosemide, and deep sedation re next in line. Lowering ICP is not the ultimate goal; preserving brain perfusion is. Thus, diuretics that lead to hypotension or excessive cerebral vasoconstriction may be counterproductive.
Epidural hematomas occur with modest trauma to the side of the heed. Patients classically have a period of unconsciousness, followed by lucid interval, and then gradual lapsing into coma again with fixed and dilated pupils and contralateral hemiparesis. CT scan will show a biconvex, lens-shaped hematoma. Craniotomy to evacuate the arterial blood is the treatment.
Intracranial venous bleeding occurs with subdural hematomas. CT scans show semilunar, crescent-shaped hematomas, and midline structures may be deviated. Some injuries require craniotomy to evacuate the venous bleeding.
MRI will not provide any more useful information then what is found on CT scan. One should only order tests for which the results provide information that may affect the course of treatment.
Placing the patient in Trendelenburg (head down) will not improve cerebral blood flow. -
Question 27 of 66
27. Question
A 14-year-old boy sustained a direct hit to the head by the puck (while playing ice-hockey) at full speed. He immediately lost consciousness and fell to the ice. The father was able to arouse him with smelling salts and get in the car. He seemed fine at first, lucid and alert to the point that his dad almost turned the car back toward the ice rink, but then he started being confused and nauseated all of a sudden, and about 20 minutes after the initial blackout he lost consciousness again. He grimaces and retracts to painful stimuli. The pupils are reactive and symmetrical bilaterally. His vital signs are within normal limits and the head CT shows lenticular extracerebral hemorrhage. What of the following is the most likely diagnosis?
Correct
Epidural hematoma
Incorrect
Explanation:
This patient has the typical clinical presentation of a rapidly evolving epidural hematoma between the dura and the cranium. These patients have a history of head trauma with loss of consciousness, then a lucid period, followed by loss of consciousness again. Clinical onset occurs over minutes to hours. Many of these injuries are associated with laceration of the middle meningeal artery. A lenticular extracerebral hemorrhage will be noted on computerized tomography of the head. The need for an operation should be determined by a neurosurgeon. Although death is a potential complication, the prognosis is good when the injury is recognized and treated.
Patients with cerebral concussion may have a history of brief (seconds to minutes) unconsciousness, then normal arousal. Disturbance of vision and equilibrium may also occur. Concussion can be divided into three grades.
In grade I the patent has confusion, but no amnesia, and no loss of consciousness
In grade II the patient has confusion and amnesia, but no loss of consciousness.
In grade III, the patient has confusion, amnesia, and loss or consciousness. The prognosis is excellent in the absences of abnormal neurologic findings.
Cerebral contusion is bruising of the brain parenchyma. The majority of the contusions occur in the frontal and temporal lobes. Multiple low-density areas and punctate hemorrhages will be noted on computerized tomography of the head. Complications may include cerebral edema and transtentorial herniation. The goal of treatment should be to treat the increased intracranial pressure. The prognosis is guarded. Munchausen syndrome is a psychiatric disease in which patients make up symptoms and simulate signs of the disease in search of attention from physicians. Multiple tests are conducted that usually show no abnormality but the patient will not accept that s/he does not have an illness, and goes from physician to physician in search of more tests, more medications, and multiple surgical procedures in search of a cure for the imaginary disease.
Subdural hematoma occurs when there is tearing of a bridging vein between the cerebral cortex and a draining venous sinus. At times, they may be caused by arterial lacerations on the brain´s surface. Patients may have history of loss of consciousness but recover.
Clinical onset occurs over hours. Computerized tomography of the head shows a crescent-shaped hemorrhage compressing the brain. Surgical evacuation is the treatment of choice. Complications include uncal herniation, focal neurologic deficits, and death. The prognosis is guarded. -
Question 28 of 66
28. Question
A 12-year-old boy has a 2-month history of recurrent headache along bitemporal deficits. Radiograph films show calcifications in the suprasellar region, and MRI images reveal a multicystic tumor displacing the optic chiasm. During surgery, the surgeon notices that the cystic cavities contain viscous yellow fluid. Which of the following is the most likely diagnosis?
Correct
Craniopharyngioma
Incorrect
Explanation:
Craniopharyngioma is a histologically benign epithelial tumor of odontogenic origin. It is typically located in the suprasellar compartment extending variably onto the sella, hypothalamus, and optic chiasm. Craniopharyngiomas usually present with endocrine or visual abnormalities. Heavy calcifications, unilocular or multilocular cysts, and viscous yellow fluid content are the classic features that allow the diagnosis on radiologic and gross examination.
Microscopic examination shows tumor composed of islands of keratinizing epithelium with multifocal calcific deposits, a picture similar to the most common of tooth-related tumors, i.e., adamantinoma (ameloblastoma). Craniopharyngiomas probably originate from misplaced odontogenic epithelium. Location of a neoplasm is often more important than histology with regard to the possibility of definite cure.
Craniopharyngioma highlights this rule. Although histologically benign, recurrences re frequent, because the tumor cannot be entirely excised because of its location close to vital structures (hypothalamus, nerves, and blood vessels). Radical surgery is usually accompanied by severe neurologic deficits.
Cysticercosis is an infestation acquired by eating improperly cooked pork. It leads to formation of (usually multiple) cysticerci, which are cystic structures lined by a characteristic three-layered epithelium and containing a clear fluid. Old cysts are calcified and degenerating cysts enhance with contrast and are accompanied by varying degrees of edema and flare. Neurocysticercosis has a propensity for the cerebral cortex and basal ganglia. Often a history of coming from an endemic area is present, and the best step in diagnosis is ELISA testing for cysticercosal antibodies.
Ganglioglioma is a neuroglial tumor composed of variable admixtures of neoplastic ganglionic (i.e., neuronal) cells and glial cells (astrocytes or oligodendrocytes). The temporal lobe is the most common location, and seizure is the most frequent clinical manifestation. Grossly, the tumor has cystic and solid components.
Pilocytic astrocytoma is a well-circumscribed, frequently cystic, low-grade astrocytoma that affects children and young adults. The two most common locations include the cerebellum and the diencephalic/hypothalamic region. It is an important differential to consider in any tumor of the suprasellar region. Pilocytic astrocytoma is rarely calcified, however, and the cysts do not contain the “machinery oil” fluid of craniopharyngioma.
Pituitary adenoma is mainly an intrasellar tumor. A large pituitary adenoma (macroadenoma) may extend into the suprasellar region, but calcifications and cysts are absent. -
Question 29 of 66
29. Question
A baby has been born with a translucent cystic structure covered by a membrane over the lower lumbosacral vertebral. Through the covering, neural tissue is visualized. He has flaccid paralysis of the lower extremities, no deep tendon reflexes, and no response to pinprick along bilateral clubfeet. A neurosurgical consult is requested. Prior to surgery, the neurosurgeon requests a CT scan of the brain to assess for which of the following conditions?
Correct
Type II Chiari malformation
Incorrect
Explanation:
The question is describing a myelomeningocele, a neural tube defect. Most are located in the lumbosacral region but may occur anywhere along the neuraxis. The location and extent of involvement will determine the major effects of the lesion. It will present as a saclike structure with a thin, translucent covering, under which may be seen neural tissue. One finds flaccid paralysis with no deep tendon reflexes of the lower extremities. There will also be no response to touch or pain and the lesion may be associated with deformations such as clubfoot or congenital hip dysplasia. The majority of patients will have a Chiari type II malformation with obstructive hydrocephalus. This must be determined prior to surgical correction because the ventricular system is shunted at the same time. With Chiari type II malformation, the fourth ventricle is elongated and the brain stem is kinked. The cerebellar vermis, pons, and medulla are displaced into the cervical canal. In type I Chiari malformation there is no associated hydrocephalus, a congenital abnormality in which the cerebellar tonsils are displaced into the cervical canal, probably due to obstruction of the fourth ventricle during development Symptoms typically do not appear until adolescence or early adulthood and consist of headache, neck pain, urinary frequency, and progressive lower-extremity spasticity.
Absence of the posterior cerebellar vermis is most commonly associated with the Dandy-Walker malformation. This is a congenital anomaly in which the roof of the fourth ventricle fails to form and results in a cystic expansion of the fourth ventricle into the posterior fossa. Most patients have hydrocephalus and many have other anomalies, particularly agenesis of the posterior cerebellar vermis.
The cystic cavity is shunted and if there is hydrocephalus, a shunt is placed into the lateral ventricle as well. Despite this, there are major neurologic problems such as developmental delay, ataxia, and long-tract motor signs.
Porencephaly is the presence of cysts within the brain resulting from developmental defects or acquired brain lesions, especially infarction. Porencephaly caused by developmental abnormalities is often associated with other brain malformations (e.g., microcephaly, encephalocele). The affected infant is generally severely neurologically compromised (including in cognitive function). -
Question 30 of 66
30. Question
A 44-year-old unemployed woman has a severe headache at the center of her head for several weeks not responding to the usual over-the-counter remedies. It is constant but worse in the mornings. No other signs or symptoms exist. It is different from the tension headaches she had in the past as they were occipital. She had modified radical mastectomy for T3, N1, M0 breast cancer 2 years ago and discontinued postoperative chemotherapy due to side effects. What is the next best step?
Correct
CT scan of the head with contrast
Incorrect
Explanation:
Persistent headache in a patient with a recent history of breast cancer (particularly node-positive) is brain metastasis until proven otherwise. Brain metastases frequently cause elevation of intracranial pressure (ICP), which produces headache that is often worse in the morning. The only acceptable course of action is to take a look (radiologically, of course), and the cheapest and most reliable way would be a CT scan of the head with contrast. Intravenous contrast material assists in identification of tumor lesions.
A CT scan of the head without contrast also could be done but is less useful in detecting tumors. Noncontrast head CT scan is usually used for detecting hamartomas or hemorrhages because blood itself appears the same as the contrast. For primary brain tumors, the MRI is favored; however, to show the presence of metastasis, an MRI is not required and a simple CT scan of the brain with contrast is adequate in most cases. Corticosteroids are used in treatment of signs and symptoms related to cerebral edema. Dexamethasone is the agent of choice for elevated ICP due to peritumoral edema. In this case, the patient´s condition appears stable. At this point the most important step is to establish the correct diagnosis, then initiate treatment. External beam radiation is indicated once the diagnosis of metastatic tumor is made with the help of imaging Studies.
External beam radiation with or without surgery is the mainstay of treating intracranial metastatic tumors.
Neurosurgery consultation may be warranted once a mass lesion is identified with the imaging studies.
Chest x-ray a reasonable choice in the workup of this patient and may identify metastatic lung masses, but CT with contrast will be the best step in management of this patient´s headache. -
Question 31 of 66
31. Question
A 7-year-old child has diplopia for the last 2 weeks. 3 months ago, he felt that his neck was getting stiffer, but for the last 2 weeks his neck is bent to one side. There is right-sided facial weakness, which includes the forehead. MRI shows marked enlargement of the pons, displacing but not occluding the fourth ventricle. Which of the following is the most likely diagnosis?
Correct
Brain-stem glioma
Incorrect
Explanation:
This patient most likely has a brain-stem glioma, as indicated by the location of the lesion in the MRI. Most patients presenting with low-grade brain-stem gliomas have a long history of minor complaints. Signs and symptoms usually reflect the tumor location and the age of the child. Most brain-stem tumors produce some neck stiffness or discomfort proceeding to torticollis, as in this patient Small brain-stem tumors usually present slowly, with long history of localizing findings such s an isolated cranial nerve deficit or contralateral hemiparesis. Medullary tumors can be associated with cranial nerve dysfunction, dysphagia, nasal speech, and apnea. Patients may also have cerebellar deficits such as nystagmus, ataxia, nausea, and vomiting. The rare focal pontine tumor can produce facial and auditory nerve dysfunction. Signs of increased intracranial pressure are uncommon.
Benign paroxysmal torticollis usually presents in the first few months of life and is a benign, self-limited condition characterized by recurrent episodes of head tilt. It is often accompanied by vomiting, pallor, irritability, ataxia, or drowsiness. Individual episodes usually last for hours, but occasionally for days. The episodes occur less frequently as the child gets older and disappear in most cases by 5 years of age. This patient started having these symptoms at a much older age, and the MRI findings clearly indicate a brain pathology.
Craniopharyngiomas occur in the suprasellar region and produce endocrine symptoms such as pan-hypopituitarism, growth failure, and diabetes insipidus. They also present with bitemporal hemianopsia (tunnel vision) due to involvement of the optic chiasma. Calcification above the sella on neuroimaging is another common feature.
Dystonic reaction can present with torticollis due to certain medications, such as phenothiazines, carbamazepine, and Phenytoin. Apart from a medication history, dystonic reaction can be ruled out because it is an acute spasm of muscle groups characterized by fixed upper gaze, neck twisting, grimace, clenched jaw, and difficulty in speech. It is mostly painful and can be frightening to the patient. The present patient has been having a stiff neck followed EY torticollis for weeks. Moreover, the MRI clearly shows a pontine tumor.
Medulloblastomas arise in the cerebellar vermis and grow down into the fourth ventricle. They commonly present with signs of increased intracranial pressure (nocturnal or morning headaches, nausea, vomiting, and altered mental status). Since the tumors typically arise in the midline cerebellar vermis, truncal ataxia, head titubation (or bobbing), nystagmus, and unsteady gait are also common, Neck stiffness and torticollis are uncommon as they are located higher up, unlike brain-stem gliomas. Cranial nerve deficits may also occur.
Medulloblastoma is the second most common brain tumor in children. Astrocytomas are the most common brain tumor in children and arise from the cerebellar hemisphere.
Meningiomas are extrinsic tumors that arise from the arachnoid cap cells. Meningiomas can arise from the dura at any site, most commonly the skull vault and at sites of dural reflection (e.g., falx cerebri, tentorium cerebelli, etc,). Other, less common sites include the optic nerve sheath and choroid plexus. Brain-stem gliomas, however, arise exclusively from the brain-stem region, as the name suggests. Moreover, meningiomas are very common in adults but rare in children. -
Question 32 of 66
32. Question
A 60-year-old hypertensive has acute onset of headache and loss of balance for 2 hours. She is conscious and oriented to person, space, and time. She feels nauseated and cannot walk or stand without help. There is paralysis of conjugated lateral gaze to the right side. There is no decrease in muscle strength or sensory loss. A CT scan of the head reveals an intracerebellar hematoma. Which is the most appropriate next step?
Correct
Immediate surgical evacuation
Incorrect
Explanation:
Hypertension is a frequent cause of intracerebral hemorrhages. Cerebellar bleeding differs from intracerebral bleeding (i.e., that occurring within the cerebral hemisphere) with respect to clinical manifestations and management. Whereas intracerebral bleeding usually develops slowly, cerebellar hematomas manifest with abrupt onset. Intracerebral bleeding usually leads to early loss of consciousness, but patients with cerebellar hematomas remain lucid until the increased pressure within the posterior fossa results in cerebellar tonsillar herniation. Cerebellar hematomas should be evacuated as soon as possible before coma ensues. In contrast to intracerebral bleeding, prompt surgical intervention may be life-saving and followed by complete or nearly complete neurologic recovery. It is thus essential to recognize this clinical syndrome promptly.
Anticoagulant treatment is contraindicated when there is evidence of intraparenchymal bleeding.
Lumbar puncture is definitely contraindicated in case of suspected intracerebellar hematoma, as it may precipitate fatal herniation of the tonsils through the foramen magnum.
MRI of the head would not add any more information compared with the CT scan in the acute stage of intracranial bleeding.
Supportive medical treatment is appropriate for most cases of hypertensive intracerebral bleeding, which develops in deep structures of the cerebral hemisphere or brainstem, i.e., basal ganglia, centrum semiovale, or pons. Measures aimed at controlling blood pressure and reducing edema are the mainstay of treatment. -
Question 33 of 66
33. Question
8 hours after trans-sphenoidal hypophysectomy for a prolactioma; a 25 year old woman becomes lethargic, confused, and eventually comatose. Exam is unremarkable except for dry mucous membranes. She received 5% dextrose in half normal saline at 100 mL per hour. Her BP is 98/72 mm Hg, pulse is 94/min, and her urinary output has averaged 600 mL per hour since operation. Which study is likely to establish the diagnosis?
Correct
Serum sodium concentration
Incorrect
Explanation:
The large urinary output at a time when she has a modest intravenous fluid intake suggests that she has developed diabetes insipidus, something that could happen with surgery in the re of the pituitary gland. The large water loss leads to rapidly developing hypernatremia, resulting in CNS symptomatology. Surgical removal of pituitary adenomas may accidently damage the anterior or posterior pituitary glands, affecting many hormones and their normal function. Observing patients closely after surgery allows for rapid diagnosis and treatment to be implemented. Blood sugar (A) is not the culprit with 5% dextrose infusing, she would develop neither hypoglycemia nor hyperglycemia of sufficient magnitude to lead to coma. BUN (B) might be elevated, because she has incurred a volume loss together with the more pernicious tonicity change, but such determination would not establish the diagnosis: it is not specific enough. CT scan of the head (C) would show brain edema when the opposite problem develops (water intoxication), but in this case it would be less diagnostic (and more expensive) than the determination of serum sodium concentration. Lumbar puncture (D) would be unrewarding. Her brain is shrunken, not swollen, and her cerebrospinal fluid would be normal.
-
Question 34 of 66
34. Question
4 days ago a 25 year old man suffered a subarachnoid hemorrhage from a ruptured berry aneurysm. The hemorrhage was evacuated, the aneurysm clipped, and he has slowly recovered and is now stable. His plasma sodium has decreased from 138 mEq/dL, to 128 mEq/dL. Fluid restriction slowed the rate of sodium decline. Urine osmolality and specific gravity are high and total urinary sodium level and fractional excretion of sodium is elevated. What is appropriate intervention if Na+ continues to decrease?
Correct
Demeclocycline
Incorrect
Explanation:
This patient has syndrome of inappropriate antidiuretic hormone release (SIADH), which can occur in patients who have head trauma or any major central nervous system procedure and also is classically associated with small cell carcinoma of the lung. This patient is clinically euvolemic and has a high urine osmolality and specific gravity, as most water is retained in the distal tubule of the kidney. The fractional excretion of sodium is high, however, as water without salt is retained. Treatment generally involves fluid restriction followed by loop diuretics (which lose relatively more water than salt) end, if necessary, sodium tablets. In patients who cannot tolerate aggressive fluid restriction for one reason or another, or in whom aggressive treatment is warranted (such as post-trauma patients already primed for seizure activity), ADH antagonists such as demeclocycline and lithium can be used. Intravenous fluids such as half normal saline (A), D5 water (B) and normal saline (E) should be avoided. The excess ADH ensures that the water administered is absorbed in the distal tubule, whereas the salt is excreted. Desmopressin (D) is an ADH analogue and would worsen this patient´s condition. It is used in the treatment of central diabetes insipidus, which results from a lack of effective ADH secretion.
-
Question 35 of 66
35. Question
A 39-year-old alcoholic man is brought to the emergency department via ambulance after had a seizure. He has had a continual, severe headache and fever for 3 weeks and a continuing sinus infection. Acetaminophen helped initially. Temperature =38.6 °C (101 .4 °F), pulse 116 beats/min, blood pressure 102/64 mm Hg. He is disoriented to time, person, and place. There is weakness of the left arm and leg. A CT scan shows:
Culture results show Bacteroides fragilis. What is the best next step in management?Correct
Metronidazole
Incorrect
Explanation:
This patient has a brain abscess. Patients who are HIV-negative who demonstrate a ring-enhancing lesion on CT head with contrast and who also have a history of headache, fever, seizures, and focal neurologic deficits will undergo a brain biopsy for culture and sensitivity testing to identify the offending bacteria. This patient also has a history of a focus of infection (a previous sinus infection from which he never fully recovered), which is itself a risk factor for the subsequent development of a brain abscess. Brain abscesses most commonly have Streptococcus in 60 to 70% of cases; however, other offending organisms can be responsible. Bacteroides accounts for 20 to 40% of brain abscesses and is an anaerobic gram- negative bacillus. All anaerobic infections in the central nervous system should be treated with intravenous metronidazole. Remember that corticosteroids should be added if substantial mass effect can be demonstrated on a scan and the mental status is significantly depressed.
Ceftazidime would be the correct answer had the culture results demonstrated gram-negative bacilli. Clindamycin should not be used for CNS infections because it does not cross the blood-brain barrier in high concentrations. Fluconazole is an antifungal and is used as prophylaxis therapy in those HIV-positive patients who are diagnosed with cryptococcal meningitis.
Penicillin would be correct had the brain biopsy demonstrated Streptococcus. Streptococcus w is the most common organism responsible for brain abscesses in HIV-negative patients. Pyrimethamine and sulfadiazine is used as empiric therapy reserved for HIV-positive patients who demonstrate a ring-enhancing lesion on CT head scan with contrast. The reason behind this is that most HIV patients who have a ring-enhancing lesion on CT scan will have either toxoplasmosis or lymphoma. If the patient improves with 10 to 14 days of pyrimethamine and sulfadiazine, then the diagnosis is toxoplasmosis and this treatment can be continued. If there is no improvement with this therapy, then the patient most likely has lymphoma.
Surgical excision would not be the first step in management. Medical treatment is first-line of therapy, and surgery should be considered in unresponsive cases. Carbenicillin is used for Pseudomonas, an aerobe. -
Question 36 of 66
36. Question
A 51-year-old man has recent onset of complex partial seizures. CT scan shows well-circumscribed, dural-based mass in the right middle cranial fossa. The tumor displaces the brain, without evidence of intraparenchymal infiltration. Diagnose:
Correct
Meningioma
Incorrect
Explanation:
The CT shows clearly an extra-axial mass (i.e., located outside the brain parenchyma). The mass pushes and displaces the underlying brain but does not infiltrate lt. It is dural-based, i.e., it develops from the dura. These featuresare characteristic of meningiomas, which are the most common benign intracranial neoplasms. These tumors originate from meningothelial cells and may grow to a large size before producing clinical symptomstology, especially when located over the cerebral convexities. On the other hand, if meningiomas develop from the meninges of the skull base, brain compression or impingement on nerves and vessels results in clinical symptoms at early stages of growth. Meningiomas may induce a reactive bone thickening in the adjacent caldarium. Craniopharyngioma develops in the suprasellar region close to the pituitary, gland, hypothalamus, and optic chiasm. Thus, the most common initial manifestations are diabetes insipidus, visual deficits, and hypopituitarism. This tumor typically occurs in children. It is often heavily calcified, which facilitates the radiologic diagnosis. Glioblastoma multiform (GBM) is the most frequent primary malignant brain tumor. It derives from astrocytes and is therefore intraaxial (i.e., grows within the brain). Grossly, it is characterized by areas of hemorrhage and necrosis. Central necrosis within the tumor results in a typical MRI/CT appearance, with a peripheral rim of contrast enhancement (ring-enhancing mass) surrounding the central dark region.
Paget disease is common bone disease that involves the skull, producing abnormal and irregular thickening of the bone. The patient may present to medical attention after noticing n increase in head size.
Schwannoma is a benign tumor of Schwann cells. The eighth cranial nerve is its most common nerve of origin, and the cerebellopontine angle is thus the most common location. Patients with schwannomas present with progressive hearing loss. If bilateral, acoustic schwannomas are typical of neurofibromatosis type 2. -
Question 37 of 66
37. Question
The CT scan in a previously healthy 28-year-old man who sustained a closed head injury in a motor vehicle accident shows hyperdense areas anterior to the temporal poles posterior to the left occipital lobe representing acute hematomas; supra-tentorial. This patient is at risk of developing which of the following forms of cerebral herniation?
Correct
Transtentorial herniation
Incorrect
Explanation:
Blood accumulating in the supratentorial compartment, as well as any rapidly growing spacs-occupying lesion in this region, may lead to displacement of the uncus over the edge of the incisura of the tentorium. This is a grave complication referred to as uncal or transtentorial herniation. The herniated uncus will compress:
- The oculomotor nerve, leading to oculomotor paralysis (manifesting with fixed and dilated pupil on the same side).
The posterior cerebral artery, leading to ipsilateral infarction of the occipital lobe - The brain stem, leading to hemorrhages within the midbrain and pons that may result in respiratory paralysis and death.
This clinical case outlines the importance of two factors in brain injuries, i.e., development of edema, and the fact that the brain is enclosed within rigid walls. Almost any pathologic process in the brain (e.g., bleeding, infarction, tumors) is associated with edema and swelling of the parenchyma, which inevitably leads to displacement of structures and compression of adjacent regions.
Cerebellar tonsillar herniation refers to downward displacement of the cerebellar tonsils through the foramen of magnum.
This results from space-occupying lesions in the infratentorial compartment, such as bleeding and tumors. It leads to compression of the medulla and death by cardiorespiratory arrest.
Reverse cerebellar herniation is a rare form of herniation due to midbrain lesions (again, hemorrhages and tumors) that push the midbrain upward through the incisura of the tentorium.
Subfalcine (cingulate) herniation describes the lateral displacement of the cingulate gyrus beneath the falx cerebri. This event is caused by space-occupying masses in the cerebral hemisphere. It leads to compression of the anterior cerebral artery and infarction of dependent cerebral territories (mostly the medial portion of the frontal and parietal lobes).
Transcalvarial herniation may develop in open (i.e., accompanied by calvarial bone fractures) head injuries if brain parenchyma is displaced outside the cranial cavity through a calvarial defect. -
Question 38 of 66
38. Question
A 76 year old male becomes “senile” over a period of 3 or 4 weeks. He used to be active and managed all of his financial activities. According to his son he fell from a horse about a week before the mental changes began. CT scan of his head would most likely show which one of the following?
Correct
Chronic subdural hematoma
Incorrect
Explanation:
People who are very old or alcoholic have smaller brains in a skull that has not changed in size; thus, very minimal trauma can make the brain “raffle around” and tear a venous sinus, from which a subdural hematoma very slowly forms. Senility does not occur in a 3 week period. Such marked changes in someone with recent trauma should trigger a search for chronic subdural hematoma. Epidural hematomas (A) are typically acute, from a tear of the middle meningeal artery following trauma. Diffuse intracerebral bleeding (C) would occur with very severe trauma and would give more acute symptoms. The frontal lobe (D) is responsible for judgment and social graces, but not for financial acumen and level of activity-the functions that this man used to have and lost over a short period of time. Brain atrophy (E) is indeed present in the very old or the alcoholic. That is what makes them prone to develop chronic subdural hematomas. But brain atrophy alone would not explain the mental changes that this man developed over a few weeks.
-
Question 39 of 66
39. Question
A 2 year old child was diagnosed with hydrocephalus soon after birth. At age 3 months he had ventricular peritoneal (VP) shunt placed. The family has been instructed for the proper care of the shunt. In last 3 days he has become irritable, complains of headache, and has developed nausea and vomiting. The family noted impairment of upward gaze in morning. His temperature is 98.6°F, BP is 111/62 mm Hg, pulse is 68/min, and respirations are 19/min. Which one is the most likely explanation for his condition?
Correct
Obstruction of the shunt
Incorrect
Explanation:
Obstruction and infections are the two main complications of VP shunts. The symptoms in this child suggest increased intracranial pressure, and the upward gaze signals ballooning of the suprapineal recess. Obstruction is the most likely cause. Obstruction can happen even when pumping is done regularly. Mechanical obstruction is the most common complication of the VP shunt and is treated by replacement of the device. Bleeding (A) is uncommon as a shunt complication. Infection of the shunt (C) produces malaise and high fever. Infection is usually with staphylococcus. Infection may or may not interfere with shunt flow. When it does, it tends to produce recurrent episodes of malfunction. Children often out grow ventricular atrial shunts, but there is more leeway with the length of an intra peritoneal catheter. Option D is therefore unlikely to explain the problem. The child has increased intracranial pressure, and he therefore still needs the shunt, which makes option E incorrect.
-
Question 40 of 66
40. Question
A 72 year old white male has a slowly enlarging, asymptomatic abdominal aortic aneurysm. As a surgeon, you should usually recommend surgical intervention when the diameter of the aneurysm approaches
Correct
5.5 cm
Incorrect
Explanation:
Based on recent clinical trials, the most common recommendation for surgical repair is when the aneurysm approached 5.5 cm in diameter. Two large studies, the Aneurysm Detection and Management (ADAM) Veteran Affairs Cooperative Study, and the United Kingdom Small Aneurysm Trial, failed to show any benefit from early surgery for men with aneurysms less than 5.5 cm in diameter. The risk of aneurysm rupture was 1% or less in both studies, with a 6 year cumulative survivals of 74% and 64%, respectively. Interestingly, the risk of aneurysm rupture was four times greater in women, indicating that 5.5 CM may be too high, but a new evidence-based threshold has not yet been defined.
-
Question 41 of 66
41. Question
An 80 year old white male has a transient ischemic attack. 85% stenosis was revealed in the affected carotid artery in a carotid duplex study. Which one of the following would be the most appropriate management at this time?
Correct
Surgical referral for carotid endarterectomy
Incorrect
Explanation:
Carotid endarterectomy is beneficial in patients with internal carotid artery stenosis of 70%-99% who have had a stroke or a transient ischemic attack attributable to the stenosis. The short-term risk of stroke is high among patients with severe stenosis of the internal carotid artery. Although data is lacking on the optimal timing of endarterectomy, when infarction is absent or limited, urgent endarterecetomy is probably indicated for patients with internal carotid artery stenosis of 70%-99% and in selected patients with stenosis of 50%-69% who can be treated surgically with a low risk of complications.
-
Question 42 of 66
42. Question
A man aged 64 year has a past history of hypertension, smoking, and hypercholesterolemia. He comes to your office after being seen in the emergency department with an episode of dysarthria and weakness on the left side of his body. Right carotid bruit is present on examination. Carotid ultrasonography reveals 70%-80% stenosis of the right carotid artery, which is confirmed by magnetic resonance angiography. Which treatment would be appropriate in addition to management of risk factors?
Correct
Carotid endarterectomy
Incorrect
Explanation:
Carotid stenosis is an important cause of transient ischemic attacks and stroke. For patients with symptomatic carotid stenosis of more than 70%, the value of carotid endarterectomy has been firmly established on the basis of three major randomized trials. Both the North American Symptomatic Carotid Endarterectomy Trial (NASET) and the European Carotid Surgery Trial (ECST) showed that only seven or eight patients would need to undergo endarterectomy to prevent one stroke in a 5-year period. The trial by the Veterans Affairs Cooperative Studies Program also showed that endarterectomy was the best treatment in this situation.
-
Question 43 of 66
43. Question
Which of the following is an indication for immediate operative intervention rather than conservative management for arterial insufficiency?
Correct
Absent pulse
Incorrect
Explanation:
Peripheral vascular disease is a manifestation of systemic atherosclerosis that leads to significant narrowing of arteries distal to the arch of the aorta. The most common symptom of peripheral vascular disease is intermittent claudication. At other times, peripheral vascular disease leads to acute or critical limb ischemia. Intermittent claudication manifests as pain in the muscles of the legs with exercise, it is experienced by 2 percent of persons older than 65 years.
Physical findings include abnormal pedal pulses, femoral artery bruit, delayed venous filling time, cool skin, and abnormal skin color. Most patients present with subtle findings and lack classic symptoms, which makes the diagnosis difficult. The standard office-based test to determine the presence of peripheral vascular disease is calculation of the ankle-brachial index.
Treatment is divided into lifestyle, medical, and surgical therapies. Lifestyle therapies focus on exercise, smoking cessation, and dietary modification. Medical therapy is directed at reducing platelet aggregation. In addition, patients with contributing disorders such as hypertension, diabetes, and hyperlipidemia need to have these conditions managed as aggressively as possible. Surgical therapies include stents, arterectomies, angioplasty, and bypass surgery, which should be considered if the patient is found to have an absent pulse. -
Question 44 of 66
44. Question
The treatment of intermittent claudication in a patient with peripheral vascular disease consists of all of the following EXCEPT
Correct
Surgical bypass.
Incorrect
Explanation:
Intermittent claudication is not an indication for surgical bypass. Rest pain should be present for surgical bypass. Exercise leads to increased collateral flow Nicotine is a vasoconstrictive agent. Patients must be meticulous about their skin and nail care to avoid non- healing ulcers. Pentoxifylline increases blood flow by a poorly understood mechanism.
-
Question 45 of 66
45. Question
A 66 year old hypertensive man has pain in the calves on walking and is relieved by rest. He does not drink alcohol, and he quit smoking 6 months ago. He is vitally stable. Exam shows an elderly man in no apparent distress. Examination of the extremities reveals decreased femoral pulses and no pedal pulses. The next best test in his management is which one of the following?
Correct
Ankle-brachial index
Incorrect
Explanation:
This clinical scenario suggests peripheral vascular disease (or arterial occlusive disease). The patient presents with claudication, which is the most common symptom of peripheral vascular disease. Ankle-brachial index (ABI) is a simple and inexpensive method that takes only minutes. ABI is determined by measurement of the systolic blood pressures (by Doppler probe) in the ankle and arm. The value obtained from the ankle is divided by the value from the arm. This ratio is referred to as the ankle-brachial index or ratio, and provides a measure of the severity of peripheral arterial disease. The normal ABI is 1.0 to 1.3, since the pressure is higher in the ankle than in the arm. Values above 1.3 suggest a non- compressible calcified vessel. An ABI lower than 0.9 has 95% sensitivity and 100% specificity for detecting angiogram-positive peripheral arterial disease and is associated with 50% stenosis in-one or more major vessels An ABI of 0.4 to 0.9 suggests a degree of arterial obstruction, often associated with claudication. An ABI below 0.4 is associated with advanced ischemia. Contrast angiography (B), duplex ultrasonography (C) and magnetic resonance angiography (D) permit the detection and location of a vascular occlusion, which is necessary when a patient is evaluated for a possible surgical procedure or angioplasty. However, these tests are not necessary in patients with the classical clinical presentation of peripheral vascular disease and abnormal ABI. Transcutaneous oximetry (E) measures tissue oxygen pressure (tension). This is frequently used in patients with wounds due to hypoxia prior to hyperbaric oxygen therapy. However, this test is not used in the diagnosis of peripheral vascular disease.
-
Question 46 of 66
46. Question
A 58 year old hypertensive man with hypercholesterolemia has thigh pain for about 6 months. It is bilateral and dull in nature, but constant with exertion. It begins after walking about 8 blocks and is relieved by rest. Ankle-brachial indices (ABIs) are performed; with both lower extremities yielding a 0.6 index. He is referred to a vascular surgeon. The next step in management is
Correct
Pulse venous recordings (PVRs)
Incorrect
Explanation:
Given his symptoms, risk factors, and ABI results, this patient likely has claudicating. Ankle-brachial indices are a ratio of the blood pressure in the ankle to the blood pressure in the arms, and are a rough but simple evaluation of lower extremity vascular path physiology. A normal vascular system distributes pressure equally to the upper and lower extremities, and therefore results in a ratio close to 1.0. An ABI of 0.7 is classically associated with claudicating, and an ABI of 0.4 is associated with rest pain. False elevation of ABIs occurs with vessel calcification, as typically seen in patients with diabetes. The general rule for vascular surgical workup is to start with least invasive testing modalities and to progress, as necessary. This will prevent unnecessary invasive testing and related complications. Accordingly, the first step in the workup of this patient is to obtain PVRs (E), which consist of Doppler ultrasound of the flow at each arterial level. This generates a flow diagram that can be compared with normal flow patterns and the level of stenosis. This is a non-invasive procedure. The next step is to characterize the identified stenos is. The gold standard is a diagnostic angiogram (B), which is maximally invasive because it requires entry to the femoral artery with a needle followed by a temporary sheath, then injection of contrast and real time fluoroscopy. This definitively identifies the location and nature of the lesion. Because the angiogram is invasive, many vascular surgeons may choose to precede this test with either CTA (A) or an MRA (D), which involves peripheral administration of contrast and thin cut imaging. This is less invasive, but also less reliable, as the timing of the contrast administration must be so perfect. Additionally, if an angiogram is being considered, the double load of contrast should be particularly harmful. Until the lesion is identified and characterized, management cannot be planned. Therefore, placement or endovascular stents (C) without further workup would clearly to an incorrect next step.
-
Question 47 of 66
47. Question
A white female, aged 38, who has had silicone breast implants for 17 years is concerned about the risk of developing joint problems from the implants. You discuss studies concerning risks of connective tissue disease in women who have silicone breast implants and
Correct
Assure her that there is no increased risk of connective tissue disease in women with silicone implants
Incorrect
Explanation:
Despite widespread publicity and legal wrangling, a large meta-analysis of women who have had silicone breast implants has produced no evidence of any significant increase in the risk for connective tissue disease in women who have these implants.
-
Question 48 of 66
48. Question
Electrosurgical destruction is contraindicated for which one of the following skin lesions?
Correct
Melanoma
Incorrect
Explanation:
Contraindications to treatment with electrosurgery include the use of pacemaker and the treatment of melanoma. All the other lesions listed can be treated with electrosurgery.
-
Question 49 of 66
49. Question
During examination of a male infant only one testicle is palpable in the scrotum. Which one of the following of true concern regarding the problem?
Correct
The risk of malignancy in the undescended testis is several times higher than in the general population
Incorrect
Explanation:
Over 4% of boys have an undescended testis at birth. The incidence is higher in premature infants. The majority of undescended testes spontaneously descend before the age of 3 months. If the testis has not descended by 6 months of age in a term infanct, it will remain undescended. Infertility and testicular malignancy are associated with undescended testes. The risk of testicular malignancy is thought to be 4-10 times higher in patients with a history of an undescended testis, and higher sill in those with bilateral undescended testes, compared to the general male population. Orchiopexy does not reduce the risk of testicu1ar cancer, which peaks between 15 and 45 years of age. Hormonal treatment with LH-RH of hCG, used in Europe, has not been shown to be effective in randomized, controlled trials. Orchiopexy between 9 and 15 months of age is the unilateral cases and 50%- 65% of bilateral undescended testes.
-
Question 50 of 66
50. Question
When performing a circumcision which of the following is contraindicated?
Correct
A penile block using 1% lidocaine (Xylocaine)/epinephrine
Incorrect
Explanation:
The only option listed that is contraindicated for circumcision is lidocaine with epinephrine. Epinephrine should NEVER be used in a local anesthetic for penile block because the penile artery is an end artery, and vasospasm can lead to necrosis of the penile tip.
-
Question 51 of 66
51. Question
A 26 year old lady visits a cosmetic surgeon for evaluation for modify the shape of her nose. She had undergone two minor surgeries on her nose in the past for cosmetic reasons. On this visit, she is accompanied by her mother, who demanded to come to the appointment with her daughter. Her mother asks the surgeon not to perform further cosmetic surgery on the patient´s nose because she believes that it looks just fine. She says that her daughter has been obsessed with the appearance of her nose and spends much of her time looking at the nose in the mirror. A feature of this disorder is which of the following?
Correct
Disruption of day to day function
Incorrect
Explanation:
The patient is most likely suffering from body dysmorphic disorder, a preoccupation with an imagined bodily defect or an exaggerated distortion of a minimal or minor defect. To be considered a mental disorder, the concern must cause clinically significant distress or impairment in social, occupational, or other important areas of functioning. Patients with this disorder are more likely to go to dermatologists and plastic surgeons than psychiatrists. Additionally, patients with his disorder have a high lifetime prevalence of depressive and anxiety disorders. This patient´s presentation is typical for body dysmorphic disorder. The cosmetic surgeon should seek psychiatric consultation.
Agoraphobia (choice A) is a fear of public/open places. It is a specific type of phobia that causes the patient to become introverted and avoid being seen in public.
Pain symptoms (choice B) are associated with somatoform pain disorder, in which a complaint of pain is unsupported by any physiologic process that can be identified by lab tests or physical examination.
Panic attacks (choice C) are a major feature of panic disorder, which is characterized by discrete periods of intense fear and discomfort with palpitations, abdominal distress, nausea, increased perspiration, chest pain, chills, and a choking sensation. Secondary gain (choice D) is an unconscious drive for medical attention, often accompanying somatoform pain disorders, somatization disorders, or conversion disorders. -
Question 52 of 66
52. Question
A 21-year-old otherwise asymptomatic male has medium- brown (cafe-au-lait) macules on the trunk and flexor creases of the elbows and knees. He has a family history of similar skin lesions. His physician suspects Recklinhausen´s disease type I. All of the following are associated with this condition EXCEPT
Correct
Acoustic neuromas.
Incorrect
Explanation:
Acoustic neuromas are associated with neurofibromatosis type II. These tumors involve bilateral eighth nerve masses and are associated with bilateral deafness. Subcutaneous nodules can be multiple or few. They are flesh-colored and can produce grotesque deformities (which are rare). These cutaneous tumors are of variable sizes and shapes, but there must be six or more with a diameter greater than 1.5 cm to make the diagnosis of neurofibromatosis. One-third of patients are asymptomatic and one-third have neurologic symptoms associated with skin lesions. Neurofibromas are tumors of Schwann cells and nerve fibroblasts which can involve spinal nerve routes that can even cause spinal cord compression. This disorder is an autosomal dominant disorder involving chromosome 17. Freckling occurs in these regions and is also common over the trunk and in the flexor creases of the elbows and knees.
-
Question 53 of 66
53. Question
Which one of the following scenarios is NOT an indication for replantation?
Correct
An adult with an ampuation of the ring finger by a lawn mower
Incorrect
Explanation:
Lawn mower injuries tend to be contaminated and macerated and are not usually appropriate for replantation. All amputations should be reattached in children. The thumb is vital for adequate hand function, and replantation should be attempted. Multiple digit amputation is also an indication for replantation.
-
Question 54 of 66
54. Question
A 2-year-old boy has had several episodes of rectal bleeding. Evaluation with a technetium-99m perfusion scan reveals a 3-cm ileal outpouching located 60 cm from the ileocecal valve. What type(s) of ectopic tissue might be present?
Correct
Both C and D
Incorrect
Explanation:
This child has a Meckel diverticulum, an ileal outpocketing typically located within 50-75 cm of the ileocecal valve. It is a congenital anomaly resulting from the persistence of the vitelline (omphalomesenteric) duct. Approximately half cause ulceration, inflammation, and gastrointestinal bleeding because of the presence of ectopic acid-secreting gastric epithelium. Pancreatic tissue may sometimes occur in these diverticula as well. Note that this is the most common type of congenital gastrointestinal anomaly.
Rule of 2´s associated with Meckel diverticulum is that: it occurs in about 2% of children, occurs within approximately 2 feet of the ileo-cecal valve, contains 2 types of ectopic mucosa (gastric and pancreatic), and its symptoms usually our by age 2.
None of the other answer choices has a relationship to Meckel diverticulum. -
Question 55 of 66
55. Question
A 5-year-old boy developed a mass 3 days ago, and it did not resolve. It is 2.5 cm in the midline immediately above the thyroid cartilage notch and elevates when he sticks his tongue out of his mouth. Diagnose:
Correct
Thyroglossal cyst
Incorrect
Explanation:
Thyroglossal duct cysts are derived from the remnant of the thyroglossal duct. The thyroglossal duct extends from the foramen cecum of the tongue base to the thyroid gland during the embryologic development of the thyroid gland, and it typically degenerates completely. If a portion of the duct remains, it can develop into a cyst and enlarge, presenting as a midline neck mass. The pathognomonic finding of a thyroglossal duct is elevation during protrusion of the tongue, which is true in this case.
Reactive lymph node, branchial cleft cyst and head and neck neoplasm would all typically present as lateral neck masses. Sometimes a thyroglossal cyst is connected to the outside by a fistulous canal, a thyroglossal fistula. Such a fistula usually arises secondarily after rupture of a cyst but may be present at birth. Therefore, choices A, B, D, and E are not correct. -
Question 56 of 66
56. Question
A 5-months-old boy is brought to the office for a mass in his left scrotum. The infant is on the examination table, quietly sucking on his pacifier. The mother states that she first noticed the mass after the baby fell from his bed 2 days ago. On examination, there is a sacculated cavity and it is possible to get above the swelling. Light easily passes through the sac. Which of the following is the most appropriate next step in management of the patient´s condition?
Correct
Doppler ultrasound
Incorrect
Explanation:
This patient probably has a hydrocele, which the mother noticed 2 days ago. Trauma often brings the area to attention. But hematocele cannot be ruled out completely because there is a history of trauma. Moreover, a transillumination test is not diagnostic for hydrocele. Transillumination may be observed with other etiologies of scrotal swelling (e.g., hernia). Bowel in a hernia can give a positive transillumination test so Doppler ultrasound needs to be done to assess perfusion, even if an acute scrotum is clinically unlikely. This must be performed urgently if there is suspicion of testicular torsion or of traumatic hemorrhage into a hydrocele or testes. Elective surgery within a week is incorrect. This would be the correct answer if the child was more than 1 year of age and there was no history of trauma, or if trauma had been ruled out using a Doppler ultrasound.
Expectant management is incorrect. Hydroceles in newborns usually resolve spontaneously by the first birthday, unless they are accompanied by an inguinal hernia. In that case, management would be supportive. But the diagnosis of hydrocele has not yet been made. Hematocele needs to be ruled out.
Fluid aspiration is not recommended because:
The hydrocele usually recurs immediately. Aspiration is associated with a risk of introducing an infection or perforating a loop of bowel if the hydrocele is associated with a hernia. Immediate surgery would be the right answer if the diagnosis was definitely acute scrotum (testicular torsion or trauma). But even though there is a history of trauma, this patient is not in pain. -
Question 57 of 66
57. Question
An infant boy has urethral meatus located on the ventral aspect of the penile shaft. Which of the following is the most correct statement about this condition?
Correct
Circumcision should be delayed
Incorrect
Explanation:
This infant has hypospadias. This condition occurs when there is incomplete invagination of ectoderm that is located at the glans penis where the normally located urethral meatus should be. When this occurs, the urethral opening is located on the ventral side of the penis, somewhere between the tip and the base of the shaft. Circumcision should never be performed directly after an infant is born with this condition. Rather, it should be delayed until approximately 6 months of age when the infant can heifer tolerate anesthesia. The reason to delay circumcision is because the prepuce will be needed for the plastic reconstruction that will need to be done.
At the time of reconstruction the infant will also undergo a circumcision.
These patients do not have an increased risk for urinary tract infections. Therefore, a urine culture is not necessary. A voiding cystourethrogram (VCUG) allows for evaluation of the bladder, urinary reflux, and the presence or absence of posterior urethral valves. None of these conditions are associated with hypospadias. VCUG does not provide any additional information about the patient´s anatomy. -
Question 58 of 66
58. Question
A 17-month-old boy undergoes an orchiopexy for a right-sided undescended abdominal testis. The left testicle was properly descended at birth. Which of the following statements about this patient´s condition is true?
Correct
Orchiopexy is primarily performed to allow palpation of the testes.
Incorrect
Explanation:
The patient described has cryptorchidism of the right testis. Cryptorchidism is associated with significantly increased risk for testicular cancer, approximately 20 times that of the general population. This risk is not only in the cryptorchid testis, because approximately 15% of testicular cancers in patients who have cryptorchidism occur in the contralateral properly descended testis. A cryptorchid testis is most commonly found in the inguinal canal, although some may be found in a pre-scrotal or intra-abdominal location. Ectopic testes located intra-abdominally have the highest risk for malignant transformation. Many undescended testes migrate into the scrotum within the first year of life. If this does not occur, then surgical relocation of the testis into the scrotum (orchiopexy) is indicated. The primary goal of orchiopexy is to move the testicle into the scrotum so that it can be easily palpated and assessed for signs of a developing testicular malignancy. Orchiectomy of the right testicle would not eliminate the risk for testicular cancer, because there is still an increased risk for malignancy in the contralateral, properly descended testis.
Orchiopexy does not allow normal fertility because patients who have cryptorchid testes typically have some degree of testicular dysgenesis in both testes, so despite repair, fertility is never completely normal. Although orchiopexy will likely be a benefit for the man psychosocially this is not the primary reason for performing the procedure. Orchiopexy has no effect on the risk for testicular cancer.
Orchidopexy may or may not diminish the risk but it does improve the prospect of early diagnosis. (Bailey) Cancer may still occur but it will be detected early as the testis will be palpable. -
Question 59 of 66
59. Question
A 6-year-old has pain around the belly button and in the right side of his abdomen along vomiting. He had bleeding per rectum at the age of 2. Vitals signs are found to be pulse of 120/min, BP of 128/86 mmHg, RR of 20/min and temp of 100.4 oF. Rebound tenderness in the RLQ is +ve along abdominal wall rigidity. Sharp pain is elicited by a voluntary cough. Labs reveal:
WBC count: 15,000/mm3 Neutrophils 70% Lymphocytes: 30% Eosinophils: 3% Basophils: 0.5% Monocytes: 5% Diagnose:
Correct
Meckel´s diverticulitis
Incorrect
Explanation:
The patient presents with the signs and symptoms similar to that of acute appendicitis. But, the diagnosis of acute appendicitis is not given as a choice. So, we have to think of conditions that mimic acute appendicitis. The differential diagnosis of acute appendicitis include Yersinia enterocolitis, mesenteric adenitis, constipation, Crohn´s disease, Meckel diverticulitis, ectopic pregnancy, ovarian torsion, etc.
Meckel´s diverticulum results from incomplete obliteration of the omphalomesenteric duct. Most patients are asymptomatic and it is mostly diagnosed incidentally when a barium study or laparotomy is performed for other abdominal conditions. In children, acute painless lower GI bleeding is the most common presenting sign due to mucosal ulceration of the ectopic gastric tissue within the diverticulum. The diagnosis is made by a Meckel scan (with the radioactive substance 99m technetium which has an affinity for gastric mucosa). Management is excision. The rule of 2-s describes this condition, well.- It occurs in 2 % of the population
- 2 times more common in males than in females
- found within 2 if of the ileocecal valve
- 2 inches long
- About 2 % develop a complication over the course of their lives.
These complications include:
- bowel obstruction
- hemorrhage
- Meckel´s diverticulitis (22%)
- umbilical fistula
Like any other diverticula, the Meckel´s diverticulum can get inflamed causing the Meckel´s Diverticulitis. The presentation of Meckel´s Diverticulitis is clinically indistinguishable from that of acute appendicitis. The correct diagnosis can often only be established at laparotomy. Only a history of rectal bleeding (which this patient had 4 years ago) may help in distinguishing this condition from appendicitis.
Other clinical findings due to peritonitis include abdominal pain in the periumbilical area that radiates to the right lower quadrant rebound tenderness, rigidity and pain on coughing.
Ascariasis is incorrect. A mass of worms can obstruct the bowel lumen (leading to acute intestinal obstruction) or even the appendicular lumen giving similar symptoms as acute appendicitis and Meckel´s diverticulitis, but ascariasis would give an eosinophilic leukocytosis picture (like any parasitic infestation). This patient has a neutrophilic leucocytosis, pointing towards a bacterial infection and away from parasitic infestation. Moreover, ascariasis would have caused some abdominal discomfort, nausea and anorexia in the past, which this patient lacks in history.
Constipation can cause right-sided abdominal pain in children and is included in the list of differentials of acute appendicitis. But the diagnosis of constipation does not explain the signs of peritonitis (tenderness, rigidity, pain on coughing) and the leukocytosis that this patient has presented with.
Even though intestinal obstruction presents with crampy abdominal pain, nausea and vomiting, and signs of peritoneal irritation, the most common symptom of obstruction is abdominal distension, which is not mentioned anywhere in this question, Moreover, the pain would not have been so localized at the umbilicus or right lower quadrant as in this patient in addition, diarrhea or constipation should been present for the diagnosis to be intestinal obstruction.
Colonic diverticulitis is a geriatric disease (never pediatric disease), mostly presenting alter the age of 60. Among old people, right-sided colonic diverticulitis is rare and can present with pain, tenderness, leucocytosis, low grade fever, etc.
Yersinia enterocolitis is another condition that closely mimics acute appendicitis, but it presents with diarrhea most commonly and it also causes reactive arthritis. Our patient neither has diarrhea, nor joint pain. Moreover, the diagnosis of Yersinia does not explain the per-rectal bleeding he had 4 years ago. -
Question 60 of 66
60. Question
A healthy looking baby girl is delivered who had breech presentation. An older sibling has significant disability from developmental dysplasia of the hip (congenital dislocation) that was not properly diagnosed. During exam the physician cannot completely abduct the thighs when the hips and knees are flexed, but a click or a snap cannot be elicited when the physician tries to manually dislocate or reduce the femoral heads. Both legs are of the same length and glutei folds are symmetrical. The most appropriate course of action is
Correct
Do ultrasound exams
Incorrect
Explanation:
When in doubt about this diagnosis, do ultrasound exams. Actually, the inability to abduct the thighs is suggestive of the problem, and the absence of other physical signs does not exclude the diagnosis. X-rays (B) will be a waste of time. There is not enough calcification at this age to see anything. Reassurance (C) or delay (D) will be catastrophic if the femoral heads are indeed growing outside of the acetabulum. Permanent disability will ensue. If the diagnosis is confirmed, the baby will be placed on a Pelvic (abduction splint or some other similar device that keeps the hips abducted and the femoral heads inside the sockets. Thus, option E where the legs are kept in adduction would be the wrong thing to do.
-
Question 61 of 66
61. Question
A 9 year old girl is brought for neck mass evaluation. A small swelling has been present since infancy, but that it has increased in size over the past few months. The mass increases in size during upper respiratory tract infections. Child had an unremarkable birth history and is otherwise presently healthy. Neck examination reveals a 2 to 3 cm midline neck mass at the level of the thyroid cartilage. It is mobile, non tender, well circumscribed, and fluctuant. The likely diagnosis is
Correct
Thyroglossal duct cyst
Incorrect
Explanation:
This is a classic presentation of a Thyroglossal duct cyst, the most common congenital, cystic, and midline neck mass. Thyroglossal duct cysts are benign cysts that can arise anywhere along the embryonic path of descent of the thyroid gland (from the foramen cecum to the level of the thyroid). Thyroglossal cysts can become infected, demonstrating rapid enlargement and tenderness, but are otherwise generally non tender and asymptomatic. Treatment is with surgical excision- The Sistrunk procedure, in which the cyst the middle third of the hyoid bone, and the thyroglossal duct tract are all excised, is associated with the lowest rate of recurrence. A cystic hygroma (A) is a congenital malformation of lymphatic tissue. These masses commonly present in the lateral neck and are composed of multiple cystic spaces of various sizes. A dermoid cyst (B) is another type of congenital neck mass found in children, but is more commonly found in the sub mental area. They also can be found in other sites of the head and neck, including the orbit, nose, nasopharynx, and oral cavity. They consist of epithelium-lined cavities filled with skin appendages (i.e., hair, hair follicles, and sebaceous glands). Lymphadenitis (C) or necrotic lymph nodes associated with respiratory infections are one of the most common cause of neck masses in infants and children. Lymphadenitis would be considered an acquired rather than a congenital neck mass, and would generally be associated with pain, fever, and an elevated white blood cell count An infected lymph node or group of “matted” nodes would likely feel firm and irregular on palpation. In addition, lymphadenopathy related to an upper respiratory infection would most commonly present in the lateral neck rather than in the midline.
-
Question 62 of 66
62. Question
A lean boy aged 7 years fell from the bicycle and sustains a deep abdominal contusion. At the ED he is found to be stable and relatively asymptomatic. CT without contrast is negative. Next day he returns with diffuse, constant abdominal pain. Abdominal exam shows generalized tenderness and muscle guarding. There is a deep ecchymotic area over the left upper quadrant where he had been hit by the handlebar. His serum amylase and lipase are elevated. Repeated CT with double contrast shows fluid accumulation in the pelvis and no free air. Which additional finding is likely to be revealed by CT?
Correct
Pancreatic injury
Incorrect
Explanation:
The tipoff is the elevated amylase and lipase. In a thin patient with no peri-pacreatic fat to outline the gland, the original CT scan could not really see the pancreas if no contrast material was used. The collection of pelvic fluid in the second CT is pancreatic juice. A duodenal blow out (A) or gastric peroration (B) should have resulted in free sir. A retroperitoneal hematoma (D) would have resulted in lower hemoglobin level and no fluid in the pelvis. Amylase and lipase should have remained normal. Blood pressure might have been lowered. Splenic laceration (E) should result in bleeding with lowered hemoglobin level, lower blood pressure, but no elevation of the pancreatic enzymes.
-
Question 63 of 66
63. Question
A premature neonate is diagnosed with patent ducts arteriosus is successfully treated with indomethacin. Oral intake is started on the third day of life, and shortly thereafter he develops feeding intolerance and abdominal distention. By 4th day he is noted to have a rapidly dropping platelet count. X-ray shows distended bowel loops throughout the abdomen. He does not have air in the billiary tree or pneumoperitoneum. Which one is the most likely diagnosis?
Correct
Necrotizing enterocolitis
Incorrect
Explanation:
Severe gastrointestinal problems that develop when a premature infant is first fed are usually caused by necrotizing enterocolitis. The air in the bowel wall is typical, and the rapidly dropping platelet count signifies the presence of sepsis. As described, there is no evidence yet of bowel perforation. The age is wrong for intussusceptions (A). That problem is typically seen at approximately age 9 months, and the presentation is intermittent colicky pain and blood tinged stools. Malrotation (B) is always a concern in newborns with abdominal problems, but the classic case that has Ladd bands presents with green vomiting and a double-bubble sign on X-rays. Meconium ileus (C) should always be suspected if the child (or the family) has cystic fibrosis. The X-ray shows a pretty classic “ground glass” appearance. Mesenteric embolus (D) is common in elderly people or those who have atrial fibrillation or a recent myocardial infarction. In the realm of abdominal catastrophes during the newborn period, it does not quite make the list of differential diagnoses.
-
Question 64 of 66
64. Question
A 2 month old boy has chronic constipation and failure to thrive. He has had infrequent bowel movements since birth. Changing to formula feed made the problem worse. He has not been gaining weight. Her pregnancy was uneventful and she delivered vaginally at full term without complications. Infant passed meconium on 3rd day and had several episodes of vomiting in the first few weeks of life. On exam His abdomen is moderately distended and non-tender to palpation. The anal sphincter tone is normal and the rectal vault is empty of stool. Which one is the appropriate next step in management?
Correct
Barium enema
Incorrect
Explanation:
This patient most likely has Hirschsprung disease, the most common cause of constipation in neonates and infants. Constipation in the neonate should be considered Hirsch sprung disease until proven otherwise when Hirsch rung disease is suspected a barium´ enema should be preformed. If Hirschsprung is present, imaging would show a mega colon with a transition zone between the normal and aganglionic segment of the colon. It is four times more common in boys then girls. Hirschsprung disease is definitively diagnosed by a mucosal biopsy and shows areas devoid of ganglion cells (E), but would not be performed before a barium enema. The aganglionic segment is usually the recto-sigmoid part of the colon. Infants may have failure to thrive and have abdominal distension. In older children with Hirsch sprung disease, the rectal vault is empty of stool. The definitive treatment of choice is surgical resection of the aganglionic segment. Functional constipation (voluntary withholding) is the most common cause of constipation outside of infancy. Constipation occurs secondary to defects in filling or emptying the rectal vault. Other causes of constipation include imperforate anus, cystic fibrosis with Meconium ileus at birth, an interiorly displaced anus, and Hirsch sprung disease. Infantile botulism may also cause constipation. Occasionally, liquid stool can pass around the obstruction and cause the false impression of diarrhea and encopresis. Treatment of functional constipation includes initial cleaning out and may involve dietary manipulation, stool softeners, and counseling. Daily enema (B) is used to accomplish bowel cleaning in patients with Hirschsprung disease awaiting surgery but that is not the most appropriate treatment as it does not lead to any lasting improvement in the patient´s bowel habits. Dietary changes (C) and stool softeners have no effect in patients with Hirschsprung disease because a section of their bowel is immobile from the lack of ganglion cells. However, they may be helpful in patients with functional constipation in whom the gastrointestinal system is functionally intact. Genetic counseling and sweat test (D) are not the most appropriate next steps in the management of this patient as the clinical presentation is not consistent with cystic fibrosis. However, they must be provided to every patient worked up for constipation in the prenatal period, as cystic fibrosis must be ruled out as a possible cause.
-
Question 65 of 66
65. Question
A 5 year old child is brought because of a 3 cm laceration in his forehead. He is crying and frightened. Practitioner decides to perform conscious sedation before suturing the laceration. Support personnel and equipment are available for vital monitoring and carrying out resuscitation if needed. Which one is appropriate to achieve a safe level of conscious sedation in this situation?
Correct
Oral or rectal midazolam or diazepam
Incorrect
Explanation:
suturing a laceration is one of the most common situations in which sedation may be required in a child. Sedation may be classified as conscious or deep. By definition, during conscious sedation the patient is able to maintain airway patency, protective airway reflexes, and responses to physical stimuli. This level of sedation is indicated for children (or adult patients) who have not fasted prior to the procedure, or patients who do not require a deep level of sedation. Nevertheless, conscious sedation should be performed by appropriately trained personnel, and only when equipment or resuscitation measures is readily available, should the need arise. For minor surgical procedures such as suturing uncomplicated linear lacerations, administration of a short acting or long acting benzodiazepine (midazolam or diazepam, respectively) by the oral or rectal route provides sufficient sedation. Intravenous access is not required, intravenous midazolam or diazepam can be used for procedures that produce more intense pain or discomfort, such as repair of complex lacerations, bone marrow aspiration, and reduction of fractures. Concomitant analgesic-sedative agents and muscle relaxants (A) is employed for deep sedation and when muscle relaxation is necessary for endotracheal intubation or other diagnostic/therapeutic procedures. Concomitant opioid and benzodiazepine administration (B) is used to achieve not only sedation, but also an adequate level of analgesia. The synergistic action of opioid and benzodiazepines increases the risk of respiratory depression. Intravenous diazepam (choice C) is an appropriate alternative to propofol. Its most common side effect is the production of visual and auditory hallucinations (about 10% of cases).
Intravenous propofol (choice D) provides rapid onset of sedation that resolves quickly once infusion is discontinued. This drug is used for procedures requiring deeper levels of sedition in appropriately fasted and stable children. -
Question 66 of 66
66. Question
An 18 months infant has has an abdominal mass. Examination shows a small bulge at the umbilicus, about 1 cm in diameter, which pops out when the girl cries. The contents of the bulge can be easily reduced. It is not painful, and the girl is otherwise asymptomatic. She is reaching all development milestones. The most appropriate next step in management is which one of the following?
Correct
No therapy unless the hernia persists beyond the age of 2 years
Incorrect
Explanation:
Small umbilical hernias can close spontaneously in children up to the age of 2 years. Thug, if a child presents before 2 years, is asymptomatic, and the hernia is not posing an immediate risk of strangulation, the physician should allow time for spontaneous resolution. Laparoscopic hernia repair (A) makes sense when the size of the incision or incisions can be significantly reduced by the use of laparoscopy (e.g., a bilateral inguinal hernia repair). In this case, however, one would need bigger incisions to introduce a laparoscope and operating instruments than one would need to directly close a 1 cm superficial defect. Elective open surgical repair (B) will be needed only if this child still has the hernia past her second birthday. Ultrasound of the abdominal mass (D) is not appropriate at this time because, umbilical hernia is a clinical diagnosis and investigations or aggressive treatment should be done only after ages 2 to 4 years. Urgent repair (E) would have been indicated if the hernia had been tender, or if the girl had been vomiting or getting distended at the same time that the hernia became irreducible.